Wage

This topic has expert replies
User avatar
Legendary Member
Posts: 1556
Joined: Tue Aug 14, 2012 11:18 pm
Thanked: 448 times
Followed by:34 members
GMAT Score:650

Wage

by theCodeToGMAT » Fri Sep 20, 2013 6:28 am
In two months, the legal minimum wage in the country of Kirlandia will increase from five Kirlandic dollars (KD5.00) Per hour to KD5.50 per hour. Opponents of this increase have argued that the resulting rise in wages will drive the inflation rate up. In fact its impact on wages will probably be negligible, since only a very small proportion of all Kirlandic workers are currently receiving less than KD5.50 per hour. Which of the following, if true, most seriously weakens the argument?

A. Most people in Kirlandia who are currently earning the minimum wage have been employed at their current jobs for less than a year.

B. Some firms in Kirlandia have paid workers considerably less than KD5.00 per hour, in violation of Kirlandic employment regulations.

C. Many businesses hire trainees at or near the minimum wage but must reward trained workers by keeping their pay levels above the pay level of trainees.

D. The greatest growth in Kirlandia's economy in recent years has been in those sectors where workers earn wages that tend to be much higher than the minimum wage.

E. The current minimum wage is insufficient for a worker holding only one job to earn enough to support a family, even when working full time at that job.


Can someone help me understand why [spoiler][E][/spoiler] wrong?
R A H U L

User avatar
Legendary Member
Posts: 643
Joined: Wed Aug 14, 2013 4:27 am
Thanked: 48 times
Followed by:7 members

by vinay1983 » Fri Sep 20, 2013 7:02 am
Is the OA C
You can, for example never foretell what any one man will do, but you can say with precision what an average number will be up to!

User avatar
Legendary Member
Posts: 1556
Joined: Tue Aug 14, 2012 11:18 pm
Thanked: 448 times
Followed by:34 members
GMAT Score:650

by theCodeToGMAT » Fri Sep 20, 2013 7:23 am
vinay1983 wrote:Is the OA C
Yes. Correct.

Can you tell me how did you eliminate choice E.
Is the problem with E that it just talks about a general case without defining how many people are indeed lying in that category? The Answer choice "C" clearly marks that by specifying "Many".
R A H U L

User avatar
Legendary Member
Posts: 643
Joined: Wed Aug 14, 2013 4:27 am
Thanked: 48 times
Followed by:7 members

by vinay1983 » Fri Sep 20, 2013 8:43 am
theCodeToGMAT wrote:In two months, the legal minimum wage in the country of Kirlandia will increase from five Kirlandic dollars (KD5.00) Per hour to KD5.50 per hour. Opponents of this increase have argued that the resulting rise in wages will drive the inflation rate up. In fact its impact on wages will probably be negligible, since only a very small proportion of all Kirlandic workers are currently receiving less than KD5.50 per hour. Which of the following, if true, most seriously weakens the argument?

Conclusion: In fact its impact on wages will probably be negligible, since only a very small proportion of all Kirlandic workers are currently receiving less than KD5.50 per hour.

So what the prompt is asking us is to find an assumption that weakens this"Rise in min wages has negligible effect idea"

Let us start analysing the options

A. Most people in Kirlandia who are currently earning the minimum wage have been employed at their current jobs for less than a year. Then the rise has no effect whatsoever. Rather this option does not help us anyway. Eliminate

B. Some firms in Kirlandia have paid workers considerably less than KD5.00 per hour, in violation of Kirlandic employment regulations. In real life maybe we can file a case against them, but as of now we are concerned about the prompt only. So we are not bothered about the wage structure here

C. Many businesses hire trainees at or near the minimum wage but must reward trained workers by keeping their pay levels above the pay level of trainees. Yes, this is something, if the business have to increase the min wages of trainees, then have to pay the experienced workers more than the minimum. This will definitely cause inflation to increase. More pay more spending. Also many gives a majority here. If many are paid more, expect increased inflation. So you are kind of correct here

D. The greatest growth in Kirlandia's economy in recent years has been in those sectors where workers earn wages that tend to be much higher than the minimum wage. Again not in the scope. Growth, we are not talking about it

E. The current minimum wage is insufficient for a worker holding only one job to earn enough to support a family, even when working full time at that job. This actually strengthens the argument. The rise is justified and is favourable. Good trap answer

Hope I could help you!
You can, for example never foretell what any one man will do, but you can say with precision what an average number will be up to!

User avatar
Legendary Member
Posts: 1556
Joined: Tue Aug 14, 2012 11:18 pm
Thanked: 448 times
Followed by:34 members
GMAT Score:650

by theCodeToGMAT » Fri Sep 20, 2013 9:07 am
Vinay, I have doubt on the explanation you gave for [E]

Question Conclusion: The impact on wages will be negligible

To weaken we need to prove: The impact on wages will NOT be negligible.


[E] The current minimum wage is insufficient for a worker holding only one job to earn enough to support a family, even when working full time at that job.
- Here, so if the wage is increased then the worker get home more money. How is this option strengthening the Conclusion??

Two possible problem which I feel are:
-> it just talks about a general case without defining how many people are indeed lying in that category
-> there is no information that wage of 5.5 will be sufficient as well.

The more I see this question the more i get confused :|
R A H U L

User avatar
GMAT Instructor
Posts: 15539
Joined: Tue May 25, 2010 12:04 pm
Location: New York, NY
Thanked: 13060 times
Followed by:1906 members
GMAT Score:790

by GMATGuruNY » Fri Sep 20, 2013 9:52 am
theCodeToGMAT wrote:In two months, the legal minimum wage in the country of Kirlandia will increase from five Kirlandic dollars (KD5.00) Per hour to KD5.50 per hour. Opponents of this increase have argued that the resulting rise in wages will drive the inflation rate up. In fact its impact on wages will probably be negligible, since only a very small proportion of all Kirlandic workers are currently receiving less than KD5.50 per hour. Which of the following, if true, most seriously weakens the argument?

A. Most people in Kirlandia who are currently earning the minimum wage have been employed at their current jobs for less than a year.

B. Some firms in Kirlandia have paid workers considerably less than KD5.00 per hour, in violation of Kirlandic employment regulations.

C. Many businesses hire trainees at or near the minimum wage but must reward trained workers by keeping their pay levels above the pay level of trainees.

D. The greatest growth in Kirlandia's economy in recent years has been in those sectors where workers earn wages that tend to be much higher than the minimum wage.

E. The current minimum wage is insufficient for a worker holding only one job to earn enough to support a family, even when working full time at that job.
The passage contends that an INCREASE IN THE MINIMUM WAGE will not drive up inflation because the increase will have a NEGLIGIBLE IMPACT ON WAGES.
To weaken the conclusion, the correct answer choice must show that the impact on wages will NOT be negligible or that inflation will be driven up IN ANOTHER WAY.

Answer choice C: Many businesses hire trainees at or near the minimum wage but must reward trained workers by keeping their pay levels above the pay level of trainees.
Thus, an increase in the minimum wage will lead to an INCREASE IN THE WAGES OF TRAINED WORKERS, weakening the conclusion that the impact on wages will be negligible.

The correct answer is C.

E. The current minimum wage is insufficient for a worker holding only one job to earn enough to support a family, even when working full time at that job.
This answer choice does not affect the link between an increase in the minimum wage and its impact on wages and inflation.
Eliminate E.
Private tutor exclusively for the GMAT and GRE, with over 20 years of experience.
Followed here and elsewhere by over 1900 test-takers.
I have worked with students based in the US, Australia, Taiwan, China, Tajikistan, Kuwait, Saudi Arabia -- a long list of countries.
My students have been admitted to HBS, CBS, Tuck, Yale, Stern, Fuqua -- a long list of top programs.

As a tutor, I don't simply teach you how I would approach problems.
I unlock the best way for YOU to solve problems.

For more information, please email me (Mitch Hunt) at [email protected].
Student Review #1
Student Review #2
Student Review #3

User avatar
Legendary Member
Posts: 643
Joined: Wed Aug 14, 2013 4:27 am
Thanked: 48 times
Followed by:7 members

by vinay1983 » Fri Sep 20, 2013 9:27 pm
GMATGuruNY wrote:
theCodeToGMAT wrote:In two months, the legal minimum wage in the country of Kirlandia will increase from five Kirlandic dollars (KD5.00) Per hour to KD5.50 per hour. Opponents of this increase have argued that the resulting rise in wages will drive the inflation rate up. In fact its impact on wages will probably be negligible, since only a very small proportion of all Kirlandic workers are currently receiving less than KD5.50 per hour. Which of the following, if true, most seriously weakens the argument?

A. Most people in Kirlandia who are currently earning the minimum wage have been employed at their current jobs for less than a year.

B. Some firms in Kirlandia have paid workers considerably less than KD5.00 per hour, in violation of Kirlandic employment regulations.

C. Many businesses hire trainees at or near the minimum wage but must reward trained workers by keeping their pay levels above the pay level of trainees.

D. The greatest growth in Kirlandia's economy in recent years has been in those sectors where workers earn wages that tend to be much higher than the minimum wage.

E. The current minimum wage is insufficient for a worker holding only one job to earn enough to support a family, even when working full time at that job.
The passage contends that an INCREASE IN THE MINIMUM WAGE will not drive up inflation because the increase will have a NEGLIGIBLE IMPACT ON WAGES.
To weaken the conclusion, the correct answer choice must show that the impact on wages will NOT be negligible or that inflation will be driven up IN ANOTHER WAY.

Answer choice C: Many businesses hire trainees at or near the minimum wage but must reward trained workers by keeping their pay levels above the pay level of trainees.
Thus, an increase in the minimum wage will lead to an INCREASE IN THE WAGES OF TRAINED WORKERS, weakening the conclusion that the impact on wages will be negligible.

The correct answer is C.

E. The current minimum wage is insufficient for a worker holding only one job to earn enough to support a family, even when working full time at that job.
This answer choice does not affect the link between an increase in the minimum wage and its impact on wages and inflation.
Eliminate E.
You took those words away from me!You are right Mitch!
You can, for example never foretell what any one man will do, but you can say with precision what an average number will be up to!

Senior | Next Rank: 100 Posts
Posts: 78
Joined: Wed May 16, 2012 8:57 pm
Thanked: 2 times

by divineacclivity » Thu Jun 05, 2014 4:12 am
GMATGuruNY wrote: The passage contends that an INCREASE IN THE MINIMUM WAGE will not drive up inflation because the increase will have a NEGLIGIBLE IMPACT ON WAGES.
To weaken the conclusion, the correct answer choice must show that the impact on wages will NOT be negligible or that inflation will be driven up IN ANOTHER WAY.

Answer choice C: Many businesses hire trainees at or near the minimum wage but must reward trained workers by keeping their pay levels above the pay level of trainees.
Thus, an increase in the minimum wage will lead to an INCREASE IN THE WAGES OF TRAINED WORKERS, weakening the conclusion that the impact on wages will be negligible.

The correct answer is C.

E. The current minimum wage is insufficient for a worker holding only one job to earn enough to support a family, even when working full time at that job.
This answer choice does not affect the link between an increase in the minimum wage and its impact on wages and inflation.
Eliminate E.
First of all, thank you for your explanation.

Second, I have a question here:
C. Many businesses hire trainees at or near the minimum wage but must reward trained workers by keeping their pay levels above the pay level of trainees.

You're saying that option C weakens because it says that since the businesses have to reward the trained workers by keeping their pay levels above those of trainees, and that's why businesses would have to increase the payouts of trained employees too if they increase the minimum payouts of trainees.
But, my thought here is, if the increase is not substantial (as the argument suggests), then the companies might not have to increase the payouts of the trained employees since the increase is v small and the argument or the option doesn't say that they have to maintain the difference between the payouts of the trained and the trainees.
Infact, in the real world scenario, that's what happens I guess. The company offers higher packages to its newer employees whereas the packages of the already working/old employees aren't increased with the same high percentage of increment.
Had the option C said that those businesses have to maintain the difference in the payouts of the trained and the trainee or have to give a uniform increment, the option would fit better as I understand.

Please help me understand the flaw in my reasoning. Many thanks.

User avatar
GMAT Instructor
Posts: 15539
Joined: Tue May 25, 2010 12:04 pm
Location: New York, NY
Thanked: 13060 times
Followed by:1906 members
GMAT Score:790

by GMATGuruNY » Thu Jun 05, 2014 8:51 am
divineacclivity wrote: Second, I have a question here:
C. Many businesses hire trainees at or near the minimum wage but must reward trained workers by keeping their pay levels above the pay level of trainees.

You're saying that option C weakens because it says that since the businesses have to reward the trained workers by keeping their pay levels above those of trainees, and that's why businesses would have to increase the payouts of trained employees too if they increase the minimum payouts of trainees.
But, my thought here is, if the increase is not substantial(as the argument suggests) , then the companies might not have to increase the payouts of the trained employees since the increase is v small and the argument or the option doesn't say that they have to maintain the difference between the payouts of the trained and the trainees.
.
The portion in red misquotes the passage.
The passage does not state that the 10% wage increase -- from KD5 to KD5.5 -- will be insubstantial.
Rather, the passage contends that the effect on wages will be negligible because ONLY A VERY SMALL PROPORTION OF WORKERS are currently receiving less than KD5.50 per hour.
According to the OA, the wages of trained workers will also have to rise, INCREASING THE PROPORTION OF WORKERS who will see a wage increase.
Thus, the conclusion that the effect on wages will be negligible is weakened.
Private tutor exclusively for the GMAT and GRE, with over 20 years of experience.
Followed here and elsewhere by over 1900 test-takers.
I have worked with students based in the US, Australia, Taiwan, China, Tajikistan, Kuwait, Saudi Arabia -- a long list of countries.
My students have been admitted to HBS, CBS, Tuck, Yale, Stern, Fuqua -- a long list of top programs.

As a tutor, I don't simply teach you how I would approach problems.
I unlock the best way for YOU to solve problems.

For more information, please email me (Mitch Hunt) at [email protected].
Student Review #1
Student Review #2
Student Review #3

Senior | Next Rank: 100 Posts
Posts: 49
Joined: Sun May 18, 2014 8:51 am

by Md Raihan Uddin » Sun Sep 21, 2014 8:08 pm
Why is D incorrect

User avatar
GMAT Instructor
Posts: 2193
Joined: Mon Feb 22, 2010 6:30 pm
Location: Vermont and Boston, MA
Thanked: 1186 times
Followed by:512 members
GMAT Score:770

by David@VeritasPrep » Mon Sep 22, 2014 12:29 am
Choice D is not moving in the correct direction at all.

Remember that you are trying to weaken the conclusion that the overall impact on wages of the increase in the minimum wage "will probably be negligible."

Now how could a law that changes the minimum wage have a negligible impact on wages overall? Well the argument says that this is true because very few people are currently earning less than the new minimum wage that will go into effect.

Now you need to therefore say that "yes, this new law will result in increased wages overall" even though only a small portion of workers make less than 5.50

Choice D talks about growth and not about wages. You would need to make your own assumptions to try to make D the correct answer.
Veritas Prep | GMAT Instructor

Veritas Prep Reviews
Save $100 off any live Veritas Prep GMAT Course

Moderator
Posts: 426
Joined: Tue Aug 22, 2017 8:48 pm
Followed by:1 members

by BTGmoderatorAT » Sun Aug 27, 2017 8:44 am
For me, I choose Letter "C"
I believed increasing minimum wages of workers is likely adjusting also the wages of the well trained workers...